\require{AMSmath}
WisFaq - de digitale vraagbaak voor wiskunde en wiskunde onderwijs


Printen

MacLaurin reeks

Van ln(1+x)

¥
å(-1)^k/k+1x^k+1
k=0

Ik wou gewoon even weten of deze klopt...

Stef
Student universiteit België - zondag 27 augustus 2006

Antwoord

Op http://mathworld.wolfram.com/MaclaurinSeries.html op #34!


zondag 27 augustus 2006

©2001-2024 WisFaq